What is the value of n?

What Is The Value Of N?

Answers

Answer 1

Answer:

C = 95 degrees

Answer 2
The answer I think would be 95 so C

Related Questions

Please answer correctly !!!! Will mark brainliest !!!!!!!

Answers

Answer:

Think it's 7.

Step-by-step explanation:

Because 5-4=1 so it have to be 7.

7+1=8

Use the diagram to estimate the perimeter, rounding each number to the nearest hundred
5,534 11
1,666 ft
1,566 ft
5,534 ft

Answers

Answer:

14,300 ft

Step-by-step explanation:

Here are the following perimeters rounded to the nearest hundred:

5534 rounds down to 5500

1666 rounds up to 1700

1566 rounds up to 1600

5534 rounds down to 5500

The sum of these is the following:

5500 ft + 1700 ft + 1600 ft + 5500 ft = 14,300 ft

Need help ASAP PLEASE WILL MARK BRAINLEST What is the value of the expression 3/6^2 +7x4-5

Answers

Answer: 277/12

Step-by-step explanation:

Use the order of operations or pemdas (parenthesis, exponents, multiplication, division, addition, subtraction) to solve this equation:

3 / 6^2 +7 x 4 - 5        

there are no parenthesis, so exponents

= 3 / 36 + 7 x 4 - 5            

now multiplication and division(from left to right)

= 3 / 36 + 7 x 4 - 5

= 1/12 + 28 - 5                  

addition and subtraction are next(it doesn't matter the order in this case):

1/12 + 23

=  1/12 + 276/12

= 277/12

i hope this helps

Which is a true statement about a 45-45-90 triangle?
O A. Each leg is 3 times as long as the hypotenuse.
O B. Each leg is 12 times as long as the hypotenuse.
O C. The hypotenuse is 3 times as long as either leg.
O D. The hypotenuse is v2 times as long as either leg.

Answers

The answer should be
D. The hypotenuse is v2 times as long as either leg.

The hypotenuse is √2 times as long as either leg of a 45-45-90 triangle

What is a Triangle?

A triangle is a plane figure or polygon with three sides and three angles.

A Triangle has three vertices and the sum of the interior angles add up to 180°

Let the Triangle be ΔABC , such that

∠A + ∠B + ∠C = 180°

The area of the triangle = ( 1/2 ) x Length x Base

For a right angle triangle

From the Pythagoras Theorem , The hypotenuse² = base² + height²

if a² + b² = c² , it is a right triangle

if a² + b² < c² , it is an obtuse triangle

if a² + b² > c² , it is an acute triangle

Given data ,

Let the triangle be represented as ΔABC

Now , the measure of ∠ACB = measure of ∠BAC = 45°

And , the measure of ∠ABC = 90°

So , the triangle is a 45-45-90 triangle

Now , the measure of the sides be AB = BC = 1 unit

And , From the Pythagoras Theorem , The hypotenuse² = base² + height²

if a² + b² = c² , it is a right triangle

The measure of side AC ( hypotenuse ) = √ ( AB )² + ( BC )²

The measure of side AC ( hypotenuse ) = √ ( 1 + 1 )

The measure of side AC ( hypotenuse ) = √2 units

Hence , the measure of the hypotenuse of 45-45-90 triangle is √2 units

To learn more about triangles click :

https://brainly.com/question/16739377

#SPJ7

You and some friends have $30. You want to order large
pizzas (p) that are $9 eachand drinks (d) that cost $1 each.
Write and graph an inequality that shows how many pizzas
and drinks can you order.?

Answers

You can buy 3 pizzas and 3 drinks

Solve the following system:
y=x + 3
2x + y = 9

Answers:
(2,5)
(5,2)
(-2,5)
(2,-5)

Answers

Answer

your anwser would be (2,5)

Step-by-step explanation:

y - x = 3 ...1

2x + y = 9 ...2

...2 - ...1

2x + y -y + x = 9 - 3

3x = 6

x = 2

replace x you will receive y value

There is a group of $5$ children, where two of the children are twins. How many ways can I distribute $6$ identical pieces of candy to the children, if the twins must get an equal amount of candy?

Answers

Answer:

50

Step-by-step explanation:

Suppose that the kids get $x_1,x_2,x_3,x_4,x_4$ pieces of candy, respectively (each twin received $x_4$ pieces). Then we seek all quadruples $(x_1,x_2,x_3,x_4)$ satisfying

\[x_1+x_2+x_3+2x_4 = 6.\]We proceed by casework, based on the number of candies the twins receive.

If $x_4 = 0$, then there are 6 candies to distribute to 3 kids. This is like arranging 6 C's and 2 |'s (dividers), so there are $\binom{8}{2} = 28$ possible distributions.

If $x_4 = 1$, there are 4 candies remaining for the other 3 kids, so there are $\binom{6}{2} = 15$ possible distributions.

If $x_4 = 2$, there are 2 candies remaining, so there are $\binom{4}{2} = 6$ possible distributions.

If $x_4 = 3$, there are 0 candies remaining, for 1 possible distribution.

Adding all of these results from the separate cases, there are a total of

\[28 + 15 + 6 + 1 = \boxed{50}\].

Shiva brought $37.00 to the art supply store. She bought a brush, a sketchbook, and a paint set. The brush was 1 3 as much as the sketchbook, and the sketchbook cost 1 2 the cost of the paint set. Shiva had $2.00 left over after buying these items. What was the cost of each item?

Answers

Answer:

b= 3.5

s= 10.5

p=21

Step-by-step explanation:

b for brush

s for sketch book

p for paint set

b=1/3s

s= 1/2 p

b+s+p=37-2

1/3 s +s+2 s =35

1/3 s +s +2 s =35

10/3 s=35

s= 35*3/10

s= 10.5

b=1/3 s = 10.5/3 = 3.5

s= 1/2 p so p= 2s = 2( 10.5) = 21

10.5+3.5 + 21 = 35

20 cm
8 cm
8 cm
20 cm
15 cm
Find the area of the arrow above.
[? ] square centimeters Irregular figures please help explain answer​

Answers

Answer:

Step-by-step explanation:

Area of the base = l x w

= 20 x 18 = 360 cm^2

Area of the triangle = hb/2

(31 x 20)/2 = 620/2 = 310 cm^2

Area of the arrow = 360 + 310

= 670 cm^2

Answer:

610 squared centimeters

Step-by-step explanation:

After seeing this video, another dog owner trained his dog, Lightning, to try to break Tillman’s skateboarding record.
Lightning’s fastest recorded time was on a 75-meter stretch where it took him 15.5 seconds. Based on these data,
did Lightning break Tillman’s record for fastest dog on a skateboard? Explain how you know.

Answers

Answer:

75/15.5 = 4.83870967742 =4.84 meters per second.

This is a faster time than 5 meters per second.

Step-by-step explanation:

Margo has 15 pounds of clay, Tim has 10 pounds of clay, Tray has 13 pounds of clay, Lila has 14 pounds of clay, and Jerry has 13 pounds of clay. How can they redistribute the clay so that they all have the same amount? Check all that apply.

Answers

Answer:

13 clay for each person

Step-by-step explanation:

Add all the clay to find the total.

15+10+13+14+13= 65 clay in total

There are 5 people in total.

Divide 65 by 5 to find the distribution for each person.

65÷5= 13

Each person get 13 clay after redistribution.

They redistribute the clay then  Each person gets 13 clay after redistribution.

We have given that,

Margo has 15 pounds of clay, Tim has 10 pounds of clay, Tray has 13 pounds of clay, Lila has 14 pounds of clay, and Jerry has 13 pounds of clay.

What is the total of the clay?

Add all the clay to find the total.

15+10+13+14+13= 65 clay in total

There are 5 people in total.

Divide 65 by 5 to find the distribution for each person.

[tex]65/5= 13[/tex]

Each person gets 13 clay after redistribution.

To learn more about the distribution visit:

https://brainly.com/question/5512053

#SPJ2

If you are given the graph of h(x) = log. x, how could you graph m(x) = log2(x+3)?
O Translate each point of the graph of h(x) 3 units up.
O Translate each point of the graph of h(x) 3 units down.
O Translate each point of the graph of h(x) 3 units right.
O Translate each point of the graph of h(x) 3 units left.

Answers

Answer:

Option (d).

Step-by-step explanation:

Note: The base of log is missing in h(x).

Consider the given functions are

[tex]h(x)=\log_2x[/tex]

[tex]m(x)=\log_2(x+3)[/tex]

The function m(x) can be written as

[tex]m(x)=h(x+3)[/tex]           ...(1)

The translation is defined as

[tex]m(x)=h(x+a)+b[/tex]           .... (2)

Where, a is horizontal shift and b is vertical shift.

If a>0, then the graph shifts a units left and if a<0, then the graph shifts a units right.

If b>0, then the graph shifts b units up and if b<0, then the graph shifts b units down.

On comparing (1) and (2), we get

[tex]a=3,b=0[/tex]

Therefore, we have to translate each point of the graph of h(x) 3 units left to get the graph of m(x).

Hence, option (d) is correct.

Ten less the quotient If number and 3 is 6

Answers

Answer:

Quotient of a number and 3 = x/3

Ten less the quotient of a number and 3 will equal 6

So...

10 - x/3 = 6

    + x/3      +x/3

10 = x/3 +6

-6              - 6

3 × 4=(x/3) × 3

12 = x

Answer:

The number is 48

Step-by-step explanation:

The number be x

Quotient of the number and 3 : x/3

Ten less the quotient of the number and x :  x/3 -10

[tex]\frac{x}{3}-10=6\\\\\frac{x}{3}=6+10\\\\\frac{x}{3}=16\\\\x=16*3\\\\x=48[/tex]

3. If you’re good at geometry could you please help me?

Answers

Answer:

Step-by-step explanation:

[tex]a^2+b^2=c^2\\14^2+x^2=c^2\\14^2=196\\39^2=1521\\x^2=1521-196\\=1325\\\sqrt{1325} \\x=36.4ft[/tex]

What is the slope of a line parallel to the line whose equation is 4x – 2y = 7?

Answers

Answer:

Use the slope-intercept form  

y = m x + b

to find the slope  m  and y-intercept  b .

Slope:  

2

y-intercept:  

− 7 2

Lines with the same slope are parallel.

parallel = 2

Answer:2

First, solve for y

-2y=7-4x

Y=2x-7/2

y=mx+b

What we really want to know is the slope(m)

Alice invests $15,000 at age 30 from the signing bonus of her new job. She hopes the investment will be worth $30,000 when she turns 40. If the interest compounds continuously, approximately what rate of growth will she need to achieve her goal? Round to the nearest tenth of a percent.

Answers

Answer:

6.9

Step-by-step explanation:

PLEASE HELP ME THIS IS URGENT I AM BEING TIMED!

Brandon entered a contest for free math materials. A total of 758 people each put in one entry, and there will be only one winner. What is the probability of Brandon winning the contest? Write your answer as a fraction. Represent a fraction using the / symbol.
Example: 1/2

Answers

Answer:

1/758

Step-by-step explanation:

Part/whole

Answer:

1/758

Step-by-step explanation:

hope that helps

A rectangular prism has a length of 9 cm a width of 5 cm and its volume is 180 cubic cm what is the height of the prism

Answers

Answer:

The height is 4cm

Step-by-step explanation:

Multiply 9 x 5 which is 45

Then divide 180 from 45

You get 4

To check your work, multiply 9 x 5 x 4 and you get 180! hopes this helps

Two bottles of water and three small bags of fruit cost a total of £2.55. If a bottle of water costs 15p more than a bag of fruit, what is the cost of a bag of fruit?

Answers

Answer:

The cost of a bag of fruit is 45p

Step-by-step explanation:

Let w be the number of water bottles.

Let f be the number of bags of fruits.

"Two bottles of water and three small bags of fruit cost a total of £2.55"

2w + 3f = 2.55

"a bottle of water costs 15p more than a bag of fruit"

w = 0.15 + f

Substitute the second formula to the first and solve.

2w + 3f = 2.55

2(0.15 + f) + 3f = 2.55

0.3 + 2f + 3f = 2.55

0.3 + 5f = 2.55

5f = 2.55 - 0.30

5f = 2.25

f = 2.25 ÷ 5

f = 0.45

Substitute to the second formula again and solve for w.

w = 0.15 + 0.45

w = 0.6

Now verify:

2w + 3f = 2.55

2(0.6) + 3(0.45) = 2.55

1.2 + 1.35 = 2.55

2.55 = 2.55 (Correct)

Click an item in the list or group of pictures at the bottom of the problem and, holding the button down, drag it into the
correct position in the answer box. Release your mouse button when the item is place. If you change your mind, drag
the item to the trashcan. Click the trashcan to clear all your answers.
Enter the equation of the circle described below.
Center (0, 0), radius = 4

Answers

The standard equation of a unit circle is

x²+y²=1

So, if we want a radius of 4, we need to square it.

so we have x²+y²=16

Answer:

x^2 + y^2 = 16

Step-by-step explanation:

The equation of a circle is given by

(x-h)^2 + (y-k)^2 = r^2  where (h,k) is the center and r is the radius

(x-0)^2 + (y-0)^2 = 4^2

x^2 + y^2 = 16

What is the value of y for the point with an x-value of 3?

Answers

Answer:

for x =3  y = 4

Step-by-step explanation:

Go across to x = 3

Then go up until you reach a dot

The got left to read the y value

for x =3  y = 4

QUICK ANSWER PLEASE The left and right page numbers of an open book are two consecutive integers whose sum is 269. Find these page numbers.

Answers

Answer:

134 and 135

Because 269/2 is 134.5, so you know that one is 134 and the other is 135.

The page numbers are 134 and 135

What are integers?

Any positive or negative number without fractions or decimal places is known as an integer, often known as a "round number" or "whole number."

Given:

The left and right page numbers of an open book are two consecutive integers whose sum is 269.

Let two consecutive integers are n, n+1.

Then,

the sum of two consecutive integers is 269.

According to the question,

n + n + 1 = 269

2n + 1 = 269

2n = 268

n = 134

The integers are 134 and 135.

Therefore,  integers are 134 and 135.

To learn more about the integers;

brainly.com/question/1768254

#SPJ2

i need help with this PLZ!!!!!!!​

Answers

Answer:

3. 91.125

4. 30.375

Step-by-step explanation:

just multiply all of the sides

MARK ME BRAINLIEST

Answer:

Given below

Step-by-step explanation:

Area of cube = s*s*s

= 4.5x4.5x4.5=

91.125 mm^3

Area of cuboid is lbh

= 6*2.25*2.25

= 30.375 mm^3

what is factored form of the expression 16st+15t

Answers

Answer:

t(16s+15)

Step-by-step explanation:

The highest common factor of 16st+15t is t.

t(16s) + t(15)

Factor.

t(16s+15)

Answer:

t ( 16s+15)

Step-by-step explanation:

=> 16st + 15t

Taking t common

=> t ( 16s+15)

A movie ticket sold for $6.50 ten years ago.
Now a movie ticket costs $9.50. What is the
percent increase?

Answers

Answer: 46%

Step-by-step explanation:

Hi, to answer this question we have to write an equation:

The price of a movie ticket ten years ago (6.50) multiplied by a percentage in decimal form (x) is equal to 9.50.

6.50x = 9.50

Solving for x  

x =9.50/6.50

x = 1.46

Multiplying by 100 to obtain the percent form:

1.46 x 100 = 146%

Finally we have to subtract 100 to obtain the percent increase:

146-100 = 46%

Feel free to ask for more if needed or if you did not understand something.

Suppose that Fx) = x3 and G(x) = -2x2. Which statement best compares the
graph of G(x) with the graph of F(x)?

Answers

Answer:

D

Step-by-step explanation:

you can just find the value of y and try some number of x

i.e. if x = 0, both of them will be 0

x = 1

G(x) result is -2

F(x) result is 1

x= -1

G(x) result will be 2

F(x) result is -1

so from these we can prove that G(x) is bigger graph than F(x) but flip vertically by y axis

What is the value of p in the equation?
4 (8-4p)+20= 12
-20
5
10

Answers

Answer:

2.5

Step-by-step explanation:

I think you're missing a value because none of those work. 2.5 does

4*2.5 = 10

8 - 10 = -2

4 (-2) = -8

-8 + 20 = 12

21. If the area of the plane shape below is
36cm, find its height.​

Answers

Answer:

4 cm

Step-by-step explanation:

The figure is a parallelogram with area (A) calculated as

A = bh ( b is the base and h the perpendicular height )

Here A = 36 and b = 9 , thus

9h = 36 ( divide both sides by 9 )

h = 4 cm

Answer:

Step-by-step explanation:

considering the breadth of the shape as 5cm and length as 9cm...

now... for finding the height of the plane shape...

we need to use the formula for area of a trapezium...

so. area of a trapezium = 1/2 x height x sum of parallel sides

we are given area of trapezium = 36cm^2

SO SUBSTITUTING THE EQUATION...

36 = 1/2 x height x (9+9)

height = 4cm

Solve the equation, –(x∕4) = 5, for x. Which of the following properties of equality did you apply? Question 7 options: A) Multiplication B) Multiplication and subtraction C) Division and addition D) Division

Answers

Answer:

A) Multiplication

The solution of given equation  x= -20

Step-by-step explanation:

Step(i):-

Given equation  - ( x /4) = 5

Multiply '-1' on both sides, we get

                     ⇒    - ( (- x /4) ) = - 5

                     ⇒    [tex]\frac{x}{4} = -5[/tex]

Step(ii):-

Multiply '4' on both sides, we get  

                     ⇒    [tex]\frac{x}{4} X 4= -5 X 4[/tex]

Cancellation '4' on left side , we get

                          x = -20

Conclusion:-

The solution of given equation  x= -20

Answer:

A

Step-by-step explanation:

The equation y = 5 represents the graph of a line perpendicular to the y-axis and passing through the point (1,5).
a) True
b) False

Answers

Answer:

False

Step-by-step explanation:

A perpendicular line parallel to the y- axis has equation

x = c

where c is the value of the x- coordinates the line passes through.

The line passes through (1, 5) with x- coordinate 1, thus

x = 1 ← equation of perpendicular line

Answer:

B. False

Step-by-step explanation:

I hope this helps!

Short version of the answer.

Other Questions
Not sure if I got it right...I know it is one of the first 2. What is the federal government's chief lawyer called? Solicitor General Chief Lawyer Attorney General Chief Justice Cynthia earns $680 in commissions and is paid $10.25 per hour. Javier earns $410 in commissions and is paid 12.50 per hour. What will you find if you solve for x in the equation 10.25x + 680 = 12.5x +410. What is the solution of 46x8 A.) x2 B.) x23 C.) x2 D.) x2 You were left $100,000 in a trust fund set up by your grandfather. The fund pays 6.5% interest. You must spend the money on your college education, and you must withdraw the money in 4 equal installments, beginning immediately. How much could you withdraw today and at the beginning of each of the next 3 years and end up with zero in the account Which of the following statements about the negative influence of stress is TRUE?A. Stress can raise your blood pressure.B. Stress puts your body in a state of exhaustion.C. Stress weakens your immune system.D. All of the above. Which of the following terms do historians and anthropologists use to refer to a group of people who share common traditions, practices, and beliefs? dynasty clan civilization culture Which statement best relates why these animals have so much blubber? Select two literary pictures that appeal to the sense of sound. Which of the following is an example of a pull? Hi All, I have some Maths Questions for you all to have a look at. Please answer - I am giving 20 points! I will mark Brainliest as well. Take care, Jake Write a specific formula to describe the variation:x varies indirectly with the cube root of y;x=3when y= 125 A young man sustains a gunshot wound to the base of his neck. He was shot point blank with a .38 caliber revolver. The entrance wound is above the left clavicle, below the level of the cricoid cartilage, and just lateral to the sternomastoid muscle. The exit wound is just above the spinous process of the right scapula. He has normal breath sounds on both sides, is awake and alert, is talking with a normal tone of voice, is neurologically intact, and is hemodynamically stable. Portable x-ray films of the neck and chest took in the emergency department showed some air in the tissues of the lower neck, but are otherwise non-diagnostic.Which of the following is the most appropriate next step in management?A. Observation for several hoursB. CT scan of the lower neck and upper chestC. Angiogram, esophagogram, esophagoscopy, and bronchoscopy prior to surgical explorationD. Immediate surgical exploration of the lower neck through a collar incisionE. Immediate surgical exploration of the upper chest through a median sternotomy Explain how mirrors can produce images that are larger or smaller than life size, as well as upright or inverted What is the distance, in units, between the points (3, -2) and (7, 5)? what is the volume of the cone below: In the word hippopotami. What fraction of the letters are Ps? Can someone please help me with this? I want to know what is 600 minutes to days 31 points!Find x.A: 8B: 8 C: 16 D: 42